LSAT and Law School Admissions Forum

Get expert LSAT preparation and law school admissions advice from PowerScore Test Preparation.

 Administrator
PowerScore Staff
  • PowerScore Staff
  • Posts: 8916
  • Joined: Feb 02, 2011
|
#47140
Complete Question Explanation
(The complete setup for this game can be found here: lsat/viewtopic.php?t=2909)

The correct answer choice is (C)

In this question, four of the answers are viable window color combinations and one is not a viable color combination. Note that the question stem contains the word “complete” in reference to the listed color combinations, meaning that the two colors listed in each answer choice are the only two colors used in the window. This use of “complete” has a significant effect on the nature of the possible answers because it limits the listed color combinations and forces you to find not just two colors that can be combined, but two colors that can be combined and be the only two colors present in a window. This restricts your ability to use hypotheticals such as the one produced in question #7. The word “complete” also appears in four of the next five questions stems, and this increases the difficulty of those questions.

This question was referenced in the setup during the discussion of the fourth rule. As established in that discussion, each window must contain either O or P, or both. Thus, to be a viable scenario, O or P must appear as one or both of the two colors. Answer choice (C), which contains neither O nor P as one of the two colors, therefore violates the fourth rule and cannot be a complete color combination in a window.
User avatar
 TootyFrooty
  • Posts: 74
  • Joined: Oct 13, 2023
|
#104690
Can you pls share why d is wrong
 Adam Tyson
PowerScore Staff
  • PowerScore Staff
  • Posts: 5153
  • Joined: Apr 14, 2011
|
#104698
D is wrong because there CAN be a window that is just purple and orange, TootyFrooty. For example, these three windows could work:

1. GPR
2. YPR
3. PO

You may be having trouble with the last rule of the game, which tells us what must occur if a window does NOT have purple. But that tells us nothing about a window that DOES have purple, and having a window with both purple and orange doesn't violate that rule!

You wouldn't be the first person to struggle initially with that type of conditional rule, where the sufficient condition is negative and the necessary condition is positive. In fact, we call it the "most dangerous conditional rule on the LSAT," and one of my favorite blog posts is about exactly that:

https://blog.powerscore.com/lsat/the-mo ... rule-lsat/

Check it out and see if that helps!
User avatar
 TootyFrooty
  • Posts: 74
  • Joined: Oct 13, 2023
|
#104740
Adam Tyson wrote: Fri Jan 05, 2024 12:17 pm D is wrong because there CAN be a window that is just purple and orange, TootyFrooty. For example, these three windows could work:

1. GPR
2. YPR
3. PO

You may be having trouble with the last rule of the game, which tells us what must occur if a window does NOT have purple. But that tells us nothing about a window that DOES have purple, and having a window with both purple and orange doesn't violate that rule!

You wouldn't be the first person to struggle initially with that type of conditional rule, where the sufficient condition is negative and the necessary condition is positive. In fact, we call it the "most dangerous conditional rule on the LSAT," and one of my favorite blog posts is about exactly that:

https://blog.powerscore.com/lsat/the-mo ... rule-lsat/

Check it out and see if that helps!
Thanks a lot, I now remember! too many concepts to keep in mind, hopefully they all eventually stick!

Get the most out of your LSAT Prep Plus subscription.

Analyze and track your performance with our Testing and Analytics Package.